Đến nội dung

HelpMeImDying

HelpMeImDying

Đăng ký: 10-11-2017
Offline Đăng nhập: 15-12-2023 - 16:15
****-

#711551 Cho x y z >0 và xyz=1. Tìm max B=$\sum \frac{1}...

Gửi bởi HelpMeImDying trong 25-06-2018 - 16:36

Có: $x^{4}+y^{4}\geq xy(x^{2}+y^{2})\Leftrightarrow (x-y)^{2}(x^{2}+xy+y^{2})\geq 0$

$\Rightarrow \sum \frac{1}{x^{4}+y^{4}+z}\leq \sum \frac{1}{xy(x^{2}+y^{2})+z}=\sum \frac{z}{x^{2}+y^{2}+z^{2}}=\frac{x+y+z}{x^{2}+y^{2}+z^{2}}\leq \frac{3}{x+y+z}\leq 1$




#711422 Cho a b c d khác1 và $a^2+b^2+c^2+d^2=1$ Tìm max P=$\fra...

Gửi bởi HelpMeImDying trong 22-06-2018 - 21:14

Nếu $a,b,c,d>0$ thì làm thế này 

Đặt $(\frac{a}{1-a};\frac{b}{1-b};\frac{c}{1-c};\frac{d}{1-d})=(x;y;z;t)\Rightarrow (a;b;c;d)=(\frac{x}{x+1};\frac{y}{y+1};\frac{z}{z+1};\frac{t}{t+1})$

Giả thiết trở thành $1=\sum (\frac{x}{x+1})^{2}$ và $P=xyzt$

Nếu $xyzt\leq 1\Rightarrow \max P=1$

Nếu $xyzt\geq 1$

Áp dụng bổ đề: $\frac{1}{(x+1)^{2}}+\frac{1}{(y+1)^{2}}\geq \frac{1}{xy+1}; \frac{1}{x^{2}+1}+\frac{1}{y^{2}+1}\geq \frac{2}{xy+1}$ với $xy\geq 1$

$1=\sum \frac{1}{(1+\frac{1}{x})^{2}}\geq \frac{1}{1+\frac{1}{xy}}+\frac{1}{1+\frac{1}{zt}}\geq \frac{2}{1+\frac{1}{\sqrt{xyzt}}}\Leftrightarrow xyzt\leq 1$$\Rightarrow xyzt=1$

Vậy $\max P=1\Leftrightarrow$ $a=b=c=d=\frac{1}{2}$




#711394 Cho a b c >0 Tìm Min P=$\frac{a}{b}+\f...

Gửi bởi HelpMeImDying trong 22-06-2018 - 14:48

Cách khác, Đặt $(\frac{a}{b};\frac{b}{c};\frac{c}{a})=(x;y;z)\Rightarrow xyz=1$

$\Rightarrow P=x+y+z+\frac{9}{x+y+z+xy+yz+zx+3}\geq x+y+z+\frac{27}{3(x+y+z)+(x+y+z)^{2}+9}$

Đặt $x+y+z=t$, ta chứng minh $P\geq 4\Leftrightarrow \frac{t^{3}+3t^{2}+9t+27}{t^{2}+3t+9}\geq 4\Leftrightarrow t^{3}-t^{2}-3t-9\geq 0\Leftrightarrow (t-3)(t^{2}+2t+3)\geq 0$ (luôn đúng do $t\geq 3$)




#710312 Đề thi vào lớp 10 chuyên Toán TP. Hà Nội năm 2018 - 2019

Gửi bởi HelpMeImDying trong 08-06-2018 - 19:06

Câu IV:

1) Có: $\widehat{FDK}=\widehat{FCA}=\widehat{FBE}$ 

và $\frac{DK}{EB}=\frac{EA}{EB}=\frac{AD}{BC}=\frac{FD}{FB}\Rightarrow \Delta FKD\sim \Delta FEB$

2,3) Có $\widehat{FKE}=\widehat{FKD}+\widehat{EKD}=\widehat{BEF}+\widehat{AEK}=\widehat{BEF}+\widehat{BEN}=\widehat{FEN}$

Gọi $G$ là trung điểm $EF$$\Rightarrow MG//FK$

$\Rightarrow \widehat{GME}=\widehat{FKE}=\widehat{FEN}$

Mặt khác, $\frac{EM}{EN}=\frac{EA}{BE}=\frac{KD}{BE}=\frac{KF}{FE}= \frac{MG}{GE}\Rightarrow \frac{MG}{GE}=\frac{GE}{EN}\Rightarrow \Delta GEM\sim \Delta GNE\Rightarrow \widehat{EGM}=\widehat{EGN}$ và $EG^{2}=GM.GN$

$\Rightarrow$ $M,N,G$ thẳng hàng hay $MN$ đi qua trung điểm $EF$ và đường thẳng $EF$ tiếp xúc với $(EMN)$

Hình gửi kèm

  • geogebra-export17.png



#710225 Đề thi tuyển sinh vào lớp 10 THPT Hà Nội năm học 2018-2019

Gửi bởi HelpMeImDying trong 07-06-2018 - 18:37

Bài V: Đặt $\sqrt{1+x}=a;\sqrt{1-x}=b\Rightarrow a\geq 1\geq b\geq 0$ và $a^{2}+b^{2}=2$

Ta có $ab\leq \frac{a^{2}+b^{2}}{2}=1$

$P=a+b+\sqrt{2(a^{2}-b^{2})}\geq 2\sqrt{ab}+\sqrt{2(a^{2}-b^{2})}\geq 2ab+\sqrt{2(a^{2}-b^{2})}$

Ta chứng minh $2ab+\sqrt{2(a^{2}-b^{2})}\geq 2=a^{2}+b^{2}\Leftrightarrow \sqrt{2(a^{2}-b^{2})}\geq (a-b)^{2}\Leftrightarrow \sqrt{a-b}(\sqrt{2(a+b)}-(a-b)\sqrt{a-b}))$ (đúng)

$\min P=2\Leftrightarrow x=0$




#710054 [TOPIC] ÔN THI BẤT ĐẲNG THỨC $\boxed{\text{THPT CHUYÊN}}$...

Gửi bởi HelpMeImDying trong 05-06-2018 - 20:27

 

$\boxed{\text{Bài 145}}$ Cho $x,y,z$ là các số thực thỏa mãn $(x-y)(x-z) =1$ ,$y \neq z$ Chứng minh 

$$\frac{1}{(x-y)^2}+\frac{1}{(y-z)^2}+\frac{1}{(z-x)^2}\geq 4$$

 

$\frac{1}{(x-y)^{2}}+\frac{1}{(y-z)^{2}}+\frac{1}{(z-x)^{2}}=\frac{1}{(x-y)^{2}}+(y-z)^{2}+(z-x)^{2}=\frac{1}{(x-y)^{2}}+(y-x)^{2}-2(y-z)(z-x)\geq 2+2=4$




#710032 [TOPIC] ÔN THI BẤT ĐẲNG THỨC $\boxed{\text{THPT CHUYÊN}}$...

Gửi bởi HelpMeImDying trong 05-06-2018 - 18:05

 

$\boxed{\text{Bài 144}}$ Cho $a,b,c$ là các số thực dương thỏa mãn $a+b+c=3$

Chứng minh $$2(ab+bc+ca) -3abc\geq a\sqrt{\frac{b^2+c^2}{2}}+b\sqrt{\frac{c^2+a^2}{2}}+c\sqrt{\frac{a^2+b^2}{2}}$$

$\sum a\sqrt{\frac{b^{2}+c^{2}}{2}}+\sum a\sqrt{bc}\leq \sum a\sqrt{2(\frac{b^{2}+c^{2}}{2}+bc)}=\sum a(b+c)=2(ab+bc+ca)$

Ta cần chứng minh: $\sqrt{abc}(\sqrt{a}+\sqrt{b}+\sqrt{c})\geq 3abc\Leftrightarrow \sqrt{a}+\sqrt{b}+\sqrt{c}\geq 3\sqrt{abc}$

Điều này đúng do: $\sqrt{a}+\sqrt{b}+\sqrt{c}=\frac{(a+b+c)(\sqrt{a}+\sqrt{b}+\sqrt{c})}{3}\geq \frac{3\sqrt[3]{abc}.3\sqrt[3]{\sqrt{abc}}}{3}= 3\sqrt{abc}$




#709967 cm ab+bc+ac<=4

Gửi bởi HelpMeImDying trong 04-06-2018 - 22:03

Giả sử $a\geq b\geq c\Rightarrow 4\leq a^{3}+3a\Leftrightarrow (a-1)(a^{2}+a+4)\geq 0\Leftrightarrow a\geq 1$

$ab+bc+ca=a(a+b+c)-a^{2}+bc=a(4-abc)-a^{2}+bc=-(a-2)^{2}+(1-a^{2})bc+4\leq 4$




#709870 ĐÈ CHUYÊN HÀ NAM 2019

Gửi bởi HelpMeImDying trong 03-06-2018 - 23:50

Câu bất: Từ gt $(xy+yz+zx)^{2}\geq (x+y+z)^{2}\geq 3(xy+yz+zx)\Rightarrow xy+yz+zx\geq 3$

Áp dụng Cauchy-Scwharz

VT$\geq \frac{(x+y+z)^{2}}{\sum \sqrt{(x+2)(x^{2}-2x+4)}}\geq \frac{(x+y+z)^{2}}{\sqrt{(x+y+z+6)(x^{2}+y^{2}+z^{2}-2(x+y+z)+12)}}\geq \frac{(x+y+z)^{2}}{(2xy+2yz+2zx+3)(x^{2}+y^{2}+z^{2}-2(x+y+z)+12))}\geq \frac{2(x+y+z)^{2}}{x^{2}+y^{2}+z^{2}+2(xy+yz+zx)-2(x+y+z)+15}=\frac{2(x+y+z)^{2}}{(x+y+z)^{2}-2(x+y+z)+15}$

Cần chứng minh $\frac{2(x+y+z)^{2}}{(x+y+z)^{2}-2(x+y+z)+15}\geq 1\Leftrightarrow (x+y+z)^{2}+2(x+y+z)-15\geq 0\Leftrightarrow (x+y+z-3)(x+y+z+5)\geq 0$ (luôn đúng do $x+y+z\geq \sqrt{3(xy+yz+zx)}\geq 3$




#709841 Đề thi KHTN môn toán chung

Gửi bởi HelpMeImDying trong 03-06-2018 - 18:11

Câu số: Xét $x+y=0\Leftrightarrow x=-y$

$\Rightarrow x-1=0\Rightarrow x=1\Rightarrow y=-1$

Xét $x+y\neq 0$, pt $\Leftrightarrow (x+y)(3x+2y)^{2}-(3x+2y)+(x+y+1)=0$

Pt có nghiệm $\Leftrightarrow \Delta\geq 0\Leftrightarrow 1-4(x+y)(x+y+1)\geq 0\Leftrightarrow (2x+2y+1)^{2}\leq 2\Leftrightarrow -\sqrt{2}\leq 2x+2y+1\leq \sqrt{2}$

Từ đó kẹp được $\begin{bmatrix} 2x+2y+1=1\\ 2x+2y+1=-1 \end{bmatrix}$ rồi thế vào giải tiếp 




#709733 [TOPIC] HÌNH HỌC ÔN THI VÀO THPT CHUYÊN 2018-2019

Gửi bởi HelpMeImDying trong 01-06-2018 - 20:36

Bài 102 :Cho tam giác ABC với AB=AC và D là trung điểm AC.Phân giác BAC cắt (BCD) tại E ở miền trong tam giác.BD cắt (ABE) tại F (Khác B).AF cắt BE tại I và CI cắt BD tại K. CMR : I là tâm đường tròn nội tiếp tam giác KAB.

Ta có: $\widehat{EBF}=\widehat{ECD}=\widehat{ABE}\Rightarrow IB$ là phân giác góc $\widehat{ABK}$

Ta chứng minh $AI$ là phân giác góc $\widehat{BAK}$

Gọi $G$ là trung điểm $BC$, $H$ là giao điểm thứ 2 của $IC$ với $(BDC)$

Do $DG//AB$$\Rightarrow \widehat{AGD}=\widehat{BAE}=\widehat{EFD}\Rightarrow$ Tứ giác $DEFG$ nội tiếp 

Có $\widehat{AFG}=\widehat{AFE}+\widehat{EFG}=\widehat{ABE}+180^{\circ}-\widehat{EDG}=\widehat{ABE}+\widehat{ADE}+\widehat{GDC}=\widehat{ABE}+\widehat{EBC}+\widehat{BAC}=\widehat{ABC}+\widehat{BAC}=180^{\circ}-\widehat{ACB}\Rightarrow$ Tứ giác $ACGF$ nội tiếp.

Lại có $IH.IC=IE.IB=IA.IF\Rightarrow$ 5 điểm $A,H,F,G,C$ cùng thuộc đường tròn đường kính $AC$$\Rightarrow DH=DC\Rightarrow BD$ là phân giác góc $\widehat{HBC}$$\Rightarrow \Delta DCK\sim \Delta DBC\Rightarrow CD^{2}=DK.DB\Rightarrow DA^{2}=DK.DB\Rightarrow \Delta DAK\sim \Delta DBA\Rightarrow \widehat{DAK}=\widehat{DBA}$

Có $\widehat{IAB}=\widehat{IEF}=\widehat{BED}-\widehat{DEF}=\widehat{DGF}-\widehat{ACG}=\widehat{DGF}-90^{\circ}+\frac{1}{2}\widehat{BAC}=\frac{1}{2}\widehat{BAC}-\frac{1}{2}\widehat{GDF}=\frac{1}{2}\widehat{BAC}-\frac{1}{2}\widehat{ABD}=\frac{1}{2}\widehat{BAC}-\frac{1}{2}\widehat{DAK}=\frac{1}{2}\widehat{BAK}\Rightarrow$$IA$ là phân giác góc $\widehat{BAK}\Rightarrow I$ là tâm đường tròn nội tiếp $\Delta BAK$

Hình gửi kèm

  • geogebra-export16.png



#709650 Đề thi tuyển sinh vào lớp 10 THPT chuyên Lê Quý Đôn, tỉnh Bà Rịa - Vũng tàu

Gửi bởi HelpMeImDying trong 31-05-2018 - 13:47

a) Có $OM \perp CM;DM\perp CM\Rightarrow O,D,M$ thẳng hàng

Do $CD//OA$$\Rightarrow \widehat{OCD}=\widehat{COA}=\widehat{DOC}\Rightarrow \Delta OCD$ cân

Có $\widehat{FEK}=\widehat{FCD}=\widehat{FOD}=\widehat{FOK}\Rightarrow$ Tứ giác $OEFK$ nội tiếp 

b) Có tứ giác $EKAC$ nội tiếp $\rightarrow BE.BC=BK.BA=\frac{BK}{CD}.BA.CD=\frac{BE}{CE}.2OM.OD=\frac{BE}{CE}.2OF.OC= \frac{BE}{CE}.OC^{2}\Rightarrow OC^{2}=CE.CB\Rightarrow \Delta OEC\sim \Delta BOC\Rightarrow \widehat{OEC}=\widehat{BOC}\Rightarrow \widehat{OEF}=\widehat{OEC}-\widehat{CEF}=\widehat{BOC}-\widehat{CDF}=180^{\circ}-\widehat{COA}-\widehat{CDF}=180^{\circ}-\widehat{FCD}-\widehat{CDF}=90^{\circ}\Rightarrow \Delta OEF\sim \Delta CED$

c) Gọi $H'$ là trung điểm $AC$ thì theo định lý Ceva đảo dễ có $AF,CK,OH'$ đồng quy.Gọi L là giao của $MH'$ với $OI$

Có $\widehat{MAH}=\widehat{MOC}=\widehat{DCF}$ $(1)$

và $\widehat{CAM}=\widehat{COD}\Rightarrow \Delta CAM\sim \Delta DCO\Rightarrow \frac{CA}{AM}=\frac{CD}{CO}\Rightarrow \frac{HA}{AM}=\frac{IC}{CO}$ $(2)$

Kết hợp $(1),(2)\Rightarrow \Delta CIO\sim \Delta AHM\Rightarrow \widehat{AHM}=\widehat{CIO}=180^{\circ}-\widehat{LOA}\Rightarrow$ Tứ giác $OAHL$ nội tiếp $\Rightarrow \widehat{OLH}=90^{\circ}\Rightarrow OI \perp HM\Rightarrow$ $H$ nằm trên đường thẳng chứa dây chung của $(O)$ với $(I)$ 

$\Rightarrow H\equiv H'(Q.E.D)$

Câu b) làm hơi dài chắc có cách ngắn hơn

Hình gửi kèm

  • geogebra-export14.png



#709648 Đề thi tuyển sinh vào lớp 10 THPT chuyên Lê Quý Đôn, tỉnh Bà Rịa - Vũng tàu

Gửi bởi HelpMeImDying trong 31-05-2018 - 12:33

Câu 3: $\frac{S}{\sqrt{2}}=\sum \frac{a}{\sqrt{(3+b)(6-2b)}}\geq \sum \frac{2a}{9-b}\geq \frac{2(a+b+c)^{2}}{9(a+b+c)-ab-bc-ca}\geq \frac{3}{4}\Leftrightarrow8(a+b+c)^{2}+3(ab+bc+ca) \geq 27(a+b+c)$

Ta có: $8(a+b+c)^{2}+9\geq 27(a+b+c) \Leftrightarrow (a+b+c-3)(8(a+b+c)-3)\geq 0$ (luôn đúng do $a+b+c\geq3$)

$\Rightarrow 8(a+b+c)^{2}+3(ab+bc+ca)\geq 8(a+b+c)^{2}+9\geq 27(a+b+c)\Rightarrow \frac{S}{\sqrt{2}}\geq \frac{3}{4}\Rightarrow S\geq \frac{3}{2\sqrt{2}}$

 




#709631 [TOPIC] HÌNH HỌC ÔN THI VÀO THPT CHUYÊN 2018-2019

Gửi bởi HelpMeImDying trong 30-05-2018 - 22:14

 

Bài 94: Từ điểm A nằm ngoài đường tròn (O) kẻ 2 tiếp tuyến AB và AC; cát tuyến ADE sao cho BD< CD; AD< AE. Gọi H là giao điểm của OA và BC. Gọi I là trung điểm DE. Kéo dài IH cắt (O) tại K sao cho H nằm giữa I và K. Gọi S là tâm đường tròn ngoại tiếp tam giác OKA. Chứng minh OS vuông góc IK.

Gọi $F$ là giao điểm thứ 2 của $IH$ với $(O)$.

Ta có: $HK.HF=HB.HC=HA.HO\Rightarrow$ Tứ giác $AFOK$ nội tiếp suy ra $KF$ là dây chung của đường tròn $(O)$ với đường tròn $(S)$

$\Rightarrow OS\perp IK$

Hình gửi kèm

  • geogebra-export13.png



#709614 Đề thi tuyển sinh vào 10 Chuyên ĐHSP Hà Nội vòng 1 2018- 2019

Gửi bởi HelpMeImDying trong 30-05-2018 - 19:07

 

 

 

Câu 5. Các số nguyên $x,x_1,x_2,...x_9$ thỏa mãn

$$(1+x_1)(1+x_2)...(1+x_9)=(1-x_1)(1-x_2)...(1-x_9)=x$$

Tính $P=x.x_1.x_2...x_9$

 

Xét $x=0\Rightarrow P=0$

Xét $P\neq 0\Rightarrow (x,x_{1},x_{2},...,x_{9})\neq 0$

Đặt $A=(x_{1}+1)(x_{2}+2)...(x_{9}+1);B=(1-x_{1})(1-x_{2})...(1-x_{9})$

Nếu $x> 0\Rightarrow A,B> 0$

Tồn tại 1 thừa số thuộc A>0. Giả sử $x_{i}+1> 0\Rightarrow 1-x_{i}< 2$

mà $x_{i}\neq 0;1\Rightarrow 1-x_{i}\neq 0;1\Rightarrow 1-x_{i}< 0\Rightarrow \frac{B}{1-x_{i}}< 0$ nên tồn tại 1 thừa số thuộc B (khác $1-x_{i}$) <0

Giả sử $1-x_{j}< 0\Rightarrow 1+x_{j}> 0\Rightarrow \frac{A}{(1+x_{i})(1+x_{j})}> 0$ nên tồn tại 1 thừa số thuộc A (khác $1+x_{i},1+x_{j}$) >0

Tiếp tục như vây suy ra tất cả thừa số trong B đều <0$\Rightarrow B< 0$$\Rightarrow$ mâu thuẫn

Lập luận tương tự với x<0 ta cũng suy ra được điều vô lí

Vậy $x=0\Rightarrow P=0$